$\Ker f^k \subseteq\Ker f^{k-1}$, violando la minimalità di $k$, \Lightning. \\
\li Dal momento che vale la decomposizione di Fitting e che $\varphi_{\restr{f}{\Ker f^k}}$ e $\varphi_{\restr{f}{\Im f^k}}$ sono coprimi tra loro (il primo è diviso solo da $t$, mentre il secondo non è diviso da $t$), $\varphi_f =\mcm(\varphi_{\restr{f}{\Ker f^k}}, \varphi_{\restr{f}{\Im f^k}})=\varphi_{\restr{f}{\Ker f^k}}\varphi_{\restr{f}{\Im f^k}}$. Si conclude quindi che $k =\mu'_a(0)$ rispetto a $\varphi_f$, ossia la molteplicità algebrica di $0$ in
tale polinomio. Analogamente si osserva che $t =\mu_a(0)$ rispetto a $p_f$, ossia la molteplicità algebrica
dell'autovalore $0$ in $f$, e quindi che $\mu_a(0)\geq k$.
dell'autovalore $0$ in $f$, e quindi che $\mu_a(0)\geq k$, \\
\li Considerando l'endomorfismo $g = f -\lambda\Id$, si osservano facilmente alcune analogie tra le proprietà
determinanti di $g$ e di $f$: $p_g(t)=\det(f -\lambda\Id- t \Id)=\det(f -(\lambda+ t)\Id)= p_f(\lambda+ t)
\implies\mu_{a, g}(0) = \mu_{a, f}(\lambda)$. Si possono dunque riscrive i precedenti risultati in termini delle
molteplicità di un generico autovalore di $f$ considerando la molteplicità di $0$ in $g$.
\end{remark}
Reiterando la decomposizione di Fitting (o applicando il teorema di decomposizione primaria), si ottiene
@ -56,8 +60,50 @@
\[ V =\Ker(f -\lambda_1\Id)^{n_1}\oplus\cdots\oplus\Ker(f -\lambda_m \Id)^{n_m}. \]
Si deduce da questa identità che $f$ è diagonalizzabile se e solo se $n_i =1$$\forall i \leq m$.
%TODO: aggiungere come proposizione e approfondire
\begin{definition}
Si definisce \textbf{autospazio generalizzato} relativo all'autovalore $\lambda_i$ di $f$, lo spazio:
\begin{remark} Riguardo alla decomposizione primaria di $V$ e agli autospazio generalizzati di $f$ si possono fare alcune osservazioni aggiuntive. \\
\li Si può riscrive la decomposizione primaria di $V$ in termini degli autospazi generalizzati di $f$ come $V =\oplus_{i=1}^m \widetilde{V_{\lambda_i}}$. \\
\li Vale in particolare che $\widetilde{V_{\lambda_i}}=\{\v\in V \mid\exists k \in\NN\mid(f-\lambda_i \Id)^k(\v)=\vec{0}\}=\bigcup_{k=0}^{\infty}\Ker(f -\lambda_i \Id)^k$, tenendo in conto la decomposizione
di Fitting e la minimalità di $n_i$. \\
\li Considerando la traslazione vista nell'ultima osservazione, si deduce che $\Ker(f -\lambda_i \Id)^{n_i}$ ammette
come unico autovalore $\lambda_i$ (separazione degli autovalori). \\
\li Poiché $f$ è diagonalizzabile se e solo se $V =\bigoplus_{i=1}^m \Ker(f -\lambda_i \Id)$, si può dedurre
un altro criterio per la diagonalizzabilità, ossia $f$ diagonalizzabile $\impliedby n_i =1$$\forall i \leq m$. \\
\li Del precedente criterio vale anche il viceversa: se $f$ è diagonalizzabile e $\lambda_1$, ..., $\lambda_k$ sono
i suoi autovalori, $V$ ammette una base di autovettori; dati allora gli indici $i_p$ che associano ogni vettore
$\vv p$ all'indice del suo rispettivo autovalore, allora sia
$\vv1^{(\lambda_{i_1})}$, ..., $\vv n ^{(\lambda_{i_n})}$ una base di $V$. Poiché $q(t)=\prod_{i=1}^k (t -\lambda_i)$ è tale che $q(f)$ si annulla in ogni vettore della base e ogni suo fattore lineare è composto da un autovalore di $f$ ed è distinto, deve valere che $\varphi_f = q$.
\end{remark}
\begin{exercise}
Si calcoli il polinomio minimo di $A =\Matrix{0&-2&0&-2&1\\1&1&0&2&1\\0&0&1&0&0\\0&0&0&-1&0\\1&2&0&2&0}$.
\end{exercise}
\begin{solution}
Innanzitutto, si calcola il polinomio caratteristico di $A$, ossia $p_A(t)=(1-t)^3(1+t)^2$, da cui si ricava
che gli autovalori di $A$ sono $1$ e $-1$, con $\mu_a(1)=3$ e $\mu_a(-1)=2$. Si può dunque decomporre $V$
come:
\[ V =\Ker(A - I)^3\oplus\Ker(A + I)^2, \]
\vskip 0.05in
e $\varphi_A$ sarà della forma $\varphi_A(t)=(t-1)^{n_1}(t+1)^{n_2}$ con $n_1\leq3$ e $n_2\leq2$.
\begin{enumerate}[(i)]
\item$\rg(A - I)=3\implies\dim\Ker(A - I)=2 < 3=\mu_a(-1)$. Si controlli adesso il rango di
$(A-I)^2$: $\rg(A - I)^2=2\implies\dim\Ker(A - I)^2=3=\mu_a(1)$, da cui $n_1=2$.
\item$\rg(A + I)=3\implies\dim\Ker(A + I)=2$. Allora, poiché $\dim\Ker(A + I)=2=\mu_a(-1)$,
si conclude che $n_2=1$.
\end{enumerate}
Quindi $\varphi_A(t)=(t-1)^2(t+1)$.
\end{solution}
\begin{exercise}
Sia $A \in M(n, \CC)$ invertibile. Dimostrare allora che se $A^3$ è diagonalizzabile, anche $A$ lo è.
@ -68,7 +114,37 @@
dove $m$ è il numero di autovalori distinti di $A^3$. Allora, detto $p(t)=\prod_{i=1}^m (t^3-\lambda_i)$, vale che
$p(A)=0$, ossia che $\varphi_A \mid p$. Dal momento che $A$ è invertibile, anche $A^3$ lo è, e quindi
$\lambda_i \neq0$$\forall i \leq m$. Poiché $p$ è allora fattorizzato in soli termini lineari distinti,
anche $\varphi_A$ deve esserlo, e quindi $A$ deve essere diagonalizzabile.
anche $\varphi_A$ deve esserlo, e quindi $A$ deve essere diagonalizzabile.\\
\end{solution}
\vskip 0.1in
Nello studio della forma canonica di Jordan è rilevante costruire una base a bandiera tale per cui la matrice
associata in tale base sia una matrice a blocchi diagonale formata da blocchi di Jordan. Si consideri
allora $g = f -\lambda\Id$, e sia $k$ la molteplicità algebrica di $\lambda$ nel polinomio minimo
di $f$ (i.e.~il $k$ minimo già visto precedentemente nella decomposizione di Fitting di $g$). \\
Si possono allora definire dei sottospazi $U_i$ secondo le seguenti decomposizioni: